« first day (34 days earlier)      last day (540 days later) » 
00:00 - 16:0016:00 - 00:00

4:00 PM
0
Q: Question about convergence of the series

billybobDoes there exist a sequence $\{n_k\}$ of positive integers such that $$\sum\limits_{k=1}^{\infty}\frac{1}{n_k}<\infty,\space \sum\limits_{k=1}^{\infty}\frac{1}{n_{n_k}}=\infty\space$$

Words such as question are uninformative in titles. Please edit the title so that it better describes the specifics of your question. Do not hesitate to make it longer or include a formula if needed. More tips here. (autocomment)Normal Human 27 secs ago
0
Q: Homology of pairs

doppzThis shows up as problem 2.2.26 in Hatcher's Algebraic Topology. Given a pair $(X,A)$ let $X\cup CA$ be $X$ with a cone $CA$ attached at $A$. Suppose that $A$ contractible in $X$. I want to show $H_n(X,A)\cong \tilde{H}_n(X)\oplus \tilde{H}_{n-1}(A)$. What I know: If $A$ contractible in $X$ th...

Short title. Homology of pairs
 
4:11 PM
0
Q: What exactly is the definition of an evaluation oracle?

user284639What is the definition of an evaluation oracle in complexity theory?

0
Q: Solving a matrix problem

TurboGiven symmetric $A,B\in\Bbb R^{n\times n}$ what is a good description of colection of $X\in\Bbb R^{n\times n}$ such that $$XX'=I$$ $$AX=XB$$ holds?

Short title. Title contains problem. Short question. Solving a matrix problem
0
Q: Express the indefinite integral $\int e^{-x^2}dx$ using function Φ(x).

George.KExpress the indefinite integral $\int e^{-x^2}dx$ using function Φ(x). Φ(x) is the following special function : Φ(x) = $1/2$ +($1/\sqrt{2π}$)$\int_0^x e^{-t^2/2}dt$

0
Q: Proving that a ring is a PID

Jean-ClaudeI posted a question earlier Ring of polynomial functions on unit hyperbola is PID but nobody responded to it and it seems like an interesting question. Maybe somebody could take a look?

Short title. Short question. Proving that a ring is a PID
0
Q: If $\limsup_{n\to\infty} \ x_{n} =-\infty$ then why $\lim_{n\to\infty} \ x_{n} =-\infty$?

Jelly BellyI cannot understand why, if $\limsup_{n\to\infty} \ x_{n} =-\infty$ then $\lim_{n\to\infty} \ x_{n} =-\infty$? Can anybody explain it? What's the relationship between $\limsup$ and $\lim$?

0
Q: 6 digit number with digit(0-9) precisely 4 or 5 different digits

KasikI just want to be sure if i'm right so i have: For 4 digits: C(10,4) = 210 - to get number of combinations of 4 different digits 1 digit repeat 2 more times - C(4,1) = 4 number of other combinations - P*(3,1,1,1) = 120 2 digit repeats 1 more time - C(4,2) = 6 number of other combinations - ...

This site uses MathJax formatting of formulas. More tips here. (from a bot)Normal Human 24 secs ago
0
Q: Intuition First and Rigor Second, or Vice Versa

user153582I've been thinking lately about the pedagogical differences in learning things informally/intuitively first vs rigorously first. It makes sense to take a few calculus courses before tackling analysis, but assuming one has a certain level of maturity (at least mid-upper level undergrad) do you thi...

0
Q: Natural deduction: $a = b \vdash f(a) = f(b)$

今天春天How can I show using natural deduction that: $a = b \vdash f(a) = f(b)$?

0
Q: Show that if A is self-adjoint and $A^{n}= I$ then $A^{2}=I$

EDUARDO> If $A$ is a self-adjoint linear transformation on the finite-dimensional inner product space such that $A^{n}=I$ for some positive integer $n$, proof that $A^2=I$.

0
Q: Proof that endomorphism in complex vector space is diagonalizable

Jean-ClaudeHow do I prove that for $V$ a complex vector space and $A\in\text{End}_{\mathbb{C}}(V)$ such that $A^m=I$ that $A$ is diagonalizable?

0
Q: Calculus - Optimization

impactI have this problem, I am not 100% sure how to figure it out. This is the problem!

Short title. Short question. Calculus - Optimization
0
Q: definition of derivative

gboxI saw that there are "two" definition of derivative: $$lim_{x\to a} \frac{f(x)-f(a)}{x-a}$$ and $$lim_{h\to 0} \frac{f(x+h)-f(x)}{h}$$ for a derivative in a specific point which definition should I use? if the two are equivalent how can I prove that $xsin(\frac{1}{x})$ is not differentiable u...

 
5:01 PM
0
Q: What does this ∩ symbol mean in terms of geometry

Planet_EarthI don't know what this symbol mean in terms of high school geometry. I've search in google for it but I was unable to find anything about it(in terms of geometry).

0
Q: Prove $\mathbb{Z}_m* \mathbb{Z}_n = \mathbb{Z}_{mn}$

George I have answered part (i) and (ii). Is there any way to do part (iii) using the first two parts?

0
Q: Dihedral Group, D4 faithful

SelfStudyFrom Artin's second edition of Algebra. Let G be the dihedral group $D_4$$ of symmetries of a square. Is the action of G on the vertices a faithful action? on the diagonals? My solution: Since every permutation of a square (except the identity permutation) will alter at least two of the vertic...

0
Q: Modes of Convergence, Real Analysis Folland Ch.2 Problem 41

Morgan WeissIf $\mu$ is $\sigma$-finite and $f_n\rightarrow f$ a.e., there exists $E_1,E_2,\ldots\subset X$ such that $\mu\left(\left(\bigcup_{1}^{\infty}E_j\right)^{c}\right)$ and $f_n\rightarrow f$ uniformly on each $E_j$. Proof: Since $\mu$ is $\sigma$-finite, then there exists an $$X = \bigcup_{1}^{\inf...

0
Q: How do I find the 30 prices of item?

CharlieMy range is .20 to $4.80 My median is 2.10 My mean is 2.10 Help me Thanks

Welcome to Math.SE, Charlie. Questions tend to get more attention when they have a tag for a broad area of mathematics relevant to the question. Some of these tags might fit. (from a bot)Normal Human 24 secs ago
0
Q: How to prove (∀x)(A→ B)→(∃x)A→(∃x)B

swordgitI am stuck on this one, I know I have to use Ax3 which is (∀x)(A→B)→(∀x)A→(∀x)B, and convert the existential quantifier to universal, but I have problem making it to become just as Ax3

0
Q: Nice way of proving that the pullback of a tensor field is smooth?

GPerezI'm going through some routine exercises in studying smooth manifolds. This one is 12.27 (d) from Lee, Introduction to Smooth Manifolds If $F:M\to N$ is smooth, and $B$ is a covariant $k$-tensor field on $N$, show that the pullback $F^*B$ is continuous, and smooth when $B$ is smooth. I thin...

0
Q: Linear maps/dual spaces

George I have done part (i) but have absolutely no idea how to do the rest. Could you provide hints. Thanks

Short title. Short question. Linear maps/dual spaces
0
Q: Is Quotient Spaces of Hausdorff also Hausdorff where the Quotient Spaces is T1?

Matan LLet X be Hausdorff space and X/~ be "~ Quotient Spaces", Given that X/~ is T1, Does it satisfy for X/~ to be Hausdorff?

0
Q: Why 1/p(H) * ▽_Θ p(H) = ▽_Θ log p(H) ???

Chao DingHere is the background of formula (the text is not important, my questions is in the formula): enter image description here Why 1/p(H) * ▽_Θ p(H) = ▽_Θ log p(H) ???

Title contains ??. Short question. Why 1/p(H) * ▽_Θ p(H) = ▽_Θ log p(H) ???
0
Q: Can four different positive integers, multiplied in pairs, equal a fifth positive integer?

Math WiseIs there a proof or counterproof of the following statement? An integer $i\in$ $Z^+$ exists such that $a*b=i$ and $c*d=i$ where $a,b,c,d\in$ $Z^+$ and $a\neq b\neq c\neq d\neq 1$ .

0
Q: Discontinuities of $f(x)=\sum \limits_{n=1}^{\infty}\dfrac{\{nx\}}{n^2}$

Raheem NajibLet $f(x)=\sum \limits_{n=1}^{\infty}\dfrac{\{nx\}}{n^2}$ where $\{\}$ is fractional part. Find all discontinuities of function $f(x)$. I think that $f(x)$ is discontinuous at every rational point. Can anyone show how to prove strictly that $f$ for example is discontinuous at $0$ or $1/2$?

Tall formulas in titles break the layout of question lists. Please replace \dfrac with \frac in the title. (autocomment)Normal Human 21 secs ago
0
Q: Contour lines/map of a function

NikolaI have the function f(x,y)=exp(-x^2-y^2) I have no idea how to find the center of the circle and the radius. I know the formula is (x-h)^2 + (y-k)^2 = r^2 but I can't reformat my equation in this form.

0
Q: Limit of sin(1/n)*n

JamgreenMy Maple input limit(sin(1/n)*n,n=infinity); says 1. I don't understand why $$ \lim_{n \to \infty} \sin\left(\frac{1}{n}\right) \cdot n = 1 $$ I know that $\lim_{n \to \infty} 1/n = 0$, so it kind of says "0 * infinity = 1". Have I overlooked some rewriting of $\sin(1/n) n$?

Short title. Limit of sin(1/n)*n
0
Q: A question on proof of the Zermelo's theorem "Every set is well-orderable."

user198613There exists some proof for the theorem. Some of them use Transfinite Recursion. Some of them use same argument with the following. Proof:(copied from proofwiki) "Let $S$ be a set. Let $\mathcal{P}(S)$ be the power set of $S$. By the Axiom of Choice, there is a choice function $c$ defined on $...

0
Q: Intersection of lines on the Prokective plane

MarkCan lines intersect more than once in the Projective plane?

0
Q: Trilinear Coordinates of schiffler point?

Yücel Yarım I know trilinear coordinates of schiffler point but I dont know how to get it. Do you have any source about it?

0
Q: Equivalence of uniform convergence in metric spaces!

Luísa BorsatoLet $f, f_1, f_2, \dots, f_n, \dots$ be continuous applications $f_i: M \rightarrow N$, $ f: M \rightarrow N$. Then, the following affirmations are equivalent: $(1)$ If $x_n \rightarrow x$ in M, then $\lim_{n \rightarrow +\infty} f_n(x_n) = f(x)$ $(2)$ $f_n \rightarrow f$ uniformly in each $K \s...

0
Q: can pythagoras theorem be applied to areas other polygons apart from squares in relation to right angular traingles??

Saha19Is the sum of area of any regular polygon made in the two sides of a right angular triangle equal to the area of the same polygon made at the hypotenuse?? if so how to prove it?the area was equal to the semi circles with diameters of the hypotenuse and the other two sides, does it apply with any ...

Welcome to Math.SE, Luísa Borsato. Consider replacing (analysis) with a more specific tag for the relevant branch of analysis. (from a bot)Normal Human 37 secs ago
0
Q: Applied Mathematics: Force of a spring

GeometryShow that the force of a spring ${\bf{{F}}}(x) = -k(x-l){\bf\vec{i}}$ has a potential $V(x)$, such that ${\bf{F}} = -\nabla V$.

0
Q: Equivalence relations, modular arithmetic, equivalence classes.

Big MikeHere is my issue. I can do modular arithmetic. I wrote a short essay on the topic. My problem comes when trying to understand equivalence classes. So in the set of positive integers, and using mod 17. Then the equivalence class of the set of positive integers mod 17, are the integers that are d...

0
Q: A problem on real analysis in which from derivative you need to tell about function.

EklavyaLet $f:(0,\infty)\to\mathbb{R}$ be differentiable.if $f'(x)\to l $ as $x\to\infty,$then show that $\frac{f(x)}{x}\to l $ as $x\to\infty$ . i have no idea where to start.any hint please

0
Q: about holomorphic function

user296431Let $U$ be an open set and $f:U\to\mathbb{C}$ be a holomorphic function with real part $u(x,y)$ and imaginary part $v(x,y)$. Is it possible that $u(x,y)^2=1+v(x,y)^3$ for $x+iy\in U$ We have Cauchy-Riemann equations. But I don't know how to use them. Can anyone give any hint? thnx for your help.

0
Q: Subscript notation logic check

AlanGiven $$\nabla\cdot(\textbf{r}\times\nabla f)~=~(\nabla\times\textbf{r})\cdot\nabla f~-~\textbf{r}\cdot(\nabla\times\nabla f)$$ I would split the equation into 2 part: $(1)~~~~~~~~~(\nabla\times\textbf{r})\cdot\nabla f = [\epsilon_{ijk}\frac{\partial x_k}{\partial x_j}]\partial_if=0\cdot\parti...

Tagged proof-theory. Subscript notation logic check
0
Q: Binomial theorem coeffecients;

1234567890Find the coefficient of x^3 in the expansion ((1+x)^3).(2+x^2)^10 I did the first part: which is expanding the second equation at x^3: (10C3).2^7.(x^2)^3 15,360(x^2)^3 but i cant figure out what to do from here..

Welcome to Math.SE, 1234567890. This site uses MathJax formatting of formulas. More tips here. (autocomment)Normal Human 21 secs ago
 
5:52 PM
-1
Q: Badge proposal: winning bounty badges

Tomasz Jakub RupI'd like to discuss proposal for having a badges related to winning bounties. Bronze - 1 winning bounty Silver - 10 winning bounties Gold - 100 winning bounties Reasons: Bounties are awarded on complicated questions Bounties has very good answers Bounties questions are most valuable

 
0
Q: How to prove ⊢(∃x)(A ^ B) ->(∃x)(A v C -> B)

swordgitThe question has a restriction that only the method of emliminating ∃ can be used.

0
Q: Series of 1/ln(n!)

slydez$\sum_{n=2}^\infty \dfrac{1}{ln(n!)}$ How do I start with this series? I can use any method to solve this problem. When I try using Ratio Test I get stuck with: $\lim\limits_{n \to \infty} \dfrac{ln(n!)}{ln((n+1)!)}= \infty$ I also tried using Comparison Test where $b_n=\frac{1}{ln(n)}$ but ...

Short title. Series of 1/ln(n!)
0
Q: Gaussian prime factorization.

Hurjui Ionuti have a hard time on factorizing elements from Z[i] in special -19+43i. I know that the primes in Z[i] are: 1+i. p from N, p=4k+3 , k integer ( p=3(mod4) ). a+bi from Z[i], p=N(a+bi)=a^2 + b^2 and p=4k+1, k integer ( p=1(mod4) ). I wonder if there is an algorithm that tells you how to fact...

This site uses MathJax formatting of formulas. More tips here. (from a bot)Normal Human 22 secs ago
0
Q: Prove there exists infinitely many real numbers $a$ such that $a(a - 3{a})$ is an integer. \{a\} = fractional part of a

Rishav Kundu Prove there exists infinitely many real numbers $a$ such that $a(a - 3\{a\})$ is an integer. {a} = fractional part of a number. Here is my proof. Are there any fallacies that I’m missing? Alternate proofs are also welcome :) Let $S$ be the set of reals obeying this property. Let $\lambda$ b...

0
Q: Can we give an ring $R$ such that every prime ideal of $R$ be maximal with $|\operatorname{Max}(R)|=\infty?$

AngelIt is well known that in commutative rings, maximal ideals are prime. Can we give an ring $R$ such that every prime ideal of $R$ be maximal with $|\operatorname{Max}(R)|=\infty?$

0
Q: Gcd of polynomial in mod 7

PersonaAI am trying to find the gcd of $$f(x)=x^{4}+5x+1$$ and $$g(x)=x^{2}-1$$ in $\mathbb{Z[x]}/7\mathbb{Z}$ To do such, I tried using Euclidian Algorithm, and first I divided f by g to get $x^{2}+1$ and a residue of $5x+2$ Then I divided $g$ by $5x+2$ to get $x^{2}-1$ and 0 residue So if I didn't...

Short title. Title ends with a digit. Gcd of polynomial in mod 7
0
Q: Finding maximum number of edges belonging to no perfect matching in a complete bipartite graph with partite set cardinality $|X|=|Y|$

298365Here's my problem: Let G be a subgraph of $K_{20,20}$. If G has a perfect matching, prove that G has at most 190 edges that belong to no perfect matching. See here for a more generalized version of this question, i.e. there're at most $n \choose2$ such edges in any $K_{n,n}$. However, I don't und...

0
Q: About the characteristic polynomial and powers of its matrix

kuhaku Let $A_{3\times 3}$ be a matrix with the characteristic polynomial $f_A(x)=(x-1)(x-2)(x-3)$, what is $f_{A^{-1}}(x)$? The eigenvalues of $A$ are $1,2,3$ and since $Av=\lambda v$ then $A^{-1}v=\lambda^{-1} v$ so the eigenvalues of $A^{-1}$ are $1,\frac 1 2, \frac 1 3$ and therefore $f_{A^{-1...

0
Q: Integration Techniques $\tan \frac{1}{2}x$

RSparkesI have recently come across the substitution $u = \tan \frac{1}{2}x$. It is said that the substitution should be used on rational functions of sin and cos. I'm wondering what exactly this means. For instance can there be sin's and cos' both in the numerator and denominator, do they have to be lin...

0
Q: Use polar coordinates to evaluate the integral

user296090Use polar coordinates, taking x=rcosθ and y=sinθ to evaluate ∬ dx dy where R is the interior of the circle x^2 +y^2 =1

0
Q: Find $g^{-1}(x)$ in terms of $f^{-1}(x)$ if $g(x)=1+f(x)$

user289293Find $g^{-1}(x)$ in terms of $f^{-1}(x)$ if $g(x)=1+f(x)$ I find it hard to operate inverse functions. So can anyone show me a detailed process?

0
Q: Given the sides of a polygon , determine if it is convex or concave

user249117We are given the sides of a polygon. We need to determine if the given polygon is convex or concave . How can this be done? What is the propery applied to determine this?

0
Q: Calculus - Optimization Question

impactYour task is to design a rectangular industrial warehouse consisting of three separate spaces of equal size. The wall materials cost 72 dollars per linear foot and your company has allocated 34560 dollars for those walls I am supposed to use the whole budget and maximize the total area. I need...

Consider adding a tag for a broader subject area to which the question belongs. Some of these tags might fit. (from a bot)Normal Human 41 secs ago
Words such as question do not add information to titles. Please edit the title so that it better describes the specifics of your question. Do not hesitate to make it longer or include a formula if needed. More tips here. (autocomment)Normal Human 38 secs ago
0
Q: $-u''+u=\delta_y$ where $u(0)=u(1)=0$

worldreporter14Let $0<y<1$ be arbitrary. What is the weak solution of the differential equation $-u''+u=\delta_y$ where $u(0)=u(1)=0$ then? The weak form of the equation above is given by $\int_0^1{u'(t)v'(t)+u(t)v(t)dt}=v(y)$ for all $v\in H_0^1((0, 1))$.

0
Q: Spanning tree help

user296451Spanning tree I've been trying to figure out this question, but no matter how many times I try to answer the question, the end result will always be a circuit. Can anyone help me?

Welcome to Math.SE, user296451. Words such as help do not add information to titles. Please edit the title so that it better describes the specifics of your question. Do not hesitate to make it longer or include a formula if needed. More tips here. (autocomment)Normal Human 48 secs ago
0
Q: Derivative $x^{ln(x)-1}$

gbox derive $x^{ln(x)-1}$ so first I have simplify the expression: $x^{ln(x)-1}=e^{ln(x^{ln(x)-1})}=e^{ln(x)\cdot (ln(x)-1)}=e^{ln^2(x)-ln(x)}$ than $(e^{ln^2(x)-ln(x)})'=e^{ln^2(x)-ln(x)}\cdot (ln^2(x)-ln(x))'=e^{ln^2(x)-ln(x)}\cdot (ln^2(x)-ln(x))'=e^{ln^2(x)-ln(x)}\cdot (\frac{2}{x}-\frac{1}...

0
Q: evalf integration with multiple free constants

Jie LiHow to get numeric value of integration: eq:=int(a+b*x^(4/5)+c*exp((1+x)^(1/7)),x=1..2) I tried to do: assume(a,real,b,real,c,real); evalf(eq,5); But it doesn't work (with output same as input). Thanks in advance,

0
Q: Continunous population models

MeryI am trying to find all equilibria and determine witch are asymptotically stable. enter image description here So can someone please help me?

Short title. Short question. Question contains please. Continunous population models
0
Q: Transitive Actions on Set

PersonaAI am preparing for test so I am posting some of my many questions, my apologizes if it is considered to many. I am wondering, Could we have a transitive action of the group $S_{4}$ acting on the set $\{1,2,3,4,5\}$? What I know is that for there to be a transitive action we would need that for...

0
Q: Find interval in which function is continuous.

farukGiven f(x) =(x²-2)/(4x²+mx+4) determine the interval of m which makes f continuous for all real numbers.

0
Q: The set of convergence sequences is a Noetherian ring

mattIt is true that the set of convergence sequences is a Noetherian ring?

0
Q: Generalized induction proof

ramana sharmaInduction question My solution for question I couldn't understand how to determine which condition to apply l+1 or m+1 in second induction step

Short title. Short question. Generalized induction proof
0
Q: $Z_{mn}$ group extension

Nex_FriedrichI am just starting a new topic in group theory and I want to understand what is a group extension. I have the following exercise (probably simple): Check that $Z_{mn}$ is an extension of $Z_{m}$ by $Z_{n}$. I want to know what is the method of solving these kind of problems.

0
Q: Two different methods give the same correct answer for the rejection region of a Poisson distribution. Are both methods valid?

BLAZEThe exam question goes as follows: A test is done to check for a predicted atomic spectral line by counting the number of photons emitted from a sample in a narrow frequency range. The hypothesis under test is that no such spectral line exists. Due to background light, the detector will also ...

0
Q: Find probability regarding Conditional probability

AllieQ: an urn contains 2 red and 3 black balls. Players 1 and 2 withdraw balls from the urn consecutively without replacement until the second red ball is selected. player 1 draws first, then player 2, and so on. Find the probability that player 1 selects the second red ball My approach: if i let P_...

This site uses MathJax formatting of formulas. More tips here. (autocomment)Normal Human 24 secs ago
0
Q: A question concerning the pointwise convergence and continuity of the shift function.

SDB1997For any a ∈ R, let f_a : R → R be the shift function defined by f_a (x) = (x−a). Show that f is continuous if and only if, whenever a sequence of real numbers {a_n} converges to zero, f_an converges pointwise to f. [I'm stuck on this problem, therefore a full solution is appreciated]

0
Q: Product measure question, using point wise limit

Morgan WeissSuppose $(X,M,\mu)$ is a measure space, and that $f$ is a real-valued function on $X\times \mathbb{R}$. Show that if the sequence $f_x$ are continuous functions for every $x\in X$ and the sections $f^{y}$ are measurable functions for every $y\in\mathbb{R}$ then $f$ is measurable with respect to t...

Words such as question are uninformative in titles. Please edit the title so that it better describes the specifics of your question. Do not hesitate to make it longer or include a formula if needed. More tips here. (from a bot)Normal Human 23 secs ago
0
Q: What is the relationship between Rings, Integral Domains, and Fields to the computational exercises at the end of each section?

SocroxI'm currently taking an abstract algebra course that I am finding challenging although very interesting. I am reviewing the sections pertaining to Rings, Integral Domains, and Fields, and am a little confused as to how the computational exercises at the end of each section relate to the concepts...

 
7:31 PM
0
Q: Guidelines for Title Formatting

GeoffreyWhat is the standard for the formatting of the titles of posts? What is the standard for editing the titles of posts? I've been away for a while, and when I logged in today I noticed that the title of one of my posts had been edited: "Spin Glass Prince Rupert's Drop" has been changed to "Spin gl...

 
0
Q: Finding a compactly supported $f$ such that $\omega = f\omega$.

Ivo Terek Let $\Omega \subseteq \Bbb R^N$ be open, $\omega = \sum_{j=1}^n \omega_j\,{\rm d}x_j$ be a $1$-form in $\Omega$ such that $\sum_{j=1}^N|\omega_j(x)|\neq 0$ for all $x \in \Omega$, and $\theta = \sum_{i=1}^n \theta_i\,{\rm d}x_i$ be another $1$-form in $\Omega$ with $\theta \wedge \omega = 0$. ...

Consider replacing (analysis) with a more specific tag for the relevant branch of analysis. (autocomment)Normal Human 24 secs ago
0
Q: theorem about equivalence of metrics, the $d$-ball is a $p$-open set

karhasI need some help on proving a theorem about topologically equivalent metrics. It seems that somehow I am missing a final but vital step on the proof, so I am going to write the theorem and represent my attempt to prove it. Theorem: Consider a non empty set $\mathbb{X}$ and two metrics $...

0
Q: Is this equality true in a triangle?

BlueSky In the triangle above : $$XY \parallel BC$$ $$XO \parallel AC$$ Now, Is it true? $$\frac{OX}{YC} = \frac{OY}{BY}$$

0
Q: MLE of B0 and B1 for Random X

DevonI'm having real trouble finding MLE of B0 and B1 for a regression estimator with a random x In other words, using E(Y|X)=B0 + B1'x and covariance sigma^2, find the MLE. I know how to plug it into the multivariate normal and stuff, but I'm having trouble solving for it.

0
Q: Let f: G to G' be a group homomorphism. Show that the set of preimages of h in G' is not a subgroup of G unless h is the neutral element in G'.

user278055Why is this true? It seems like the inverse of an image should be in G and retain all the properties of a group.

0
Q: Quasi-subadditive sequence

billybobFind the maximum $c\in[1,+\infty)$ such that for every sequence $\{x_k\}$ satisfying $\space0\leqslant x_{n+m}\leqslant c(x_n+x_m),\space\space\forall m, n\in\mathbb{N}\space\space$ it follows that $\space\exists\lim\limits_{n\to\infty}\frac{x_n}{n^c}<\infty$.

0
Q: derivative of $10^xlog_{10}(x)$

gbox derive $10^xlog_{10}(x)$ $10^x\cdot ln(10)\cdot log_{10}(x)+\frac{1}{x\cdot ln(10)}$ Is it right?

Short title. Short question. derivative of $10^xlog_{10}(x)$
0
Q: n is a prime and then (n-1)!+1 is divisible by n. How to prove that?

Richardn is a prime and then (n-1)!+1 is divisible by n. How to prove that? Hint: [Zn{0},*n] is a cyclic group of order (n-1)

0
Q: Abstract Algebra, group theory

Rahul12It is given that order of exactly eight elements of group G is 3. We have to find the number of subgroups of order 3?

 
8:18 PM
0
Q: Can somebody please help me with this probability probme involving cars?

Joseph RishikThe time it takes to service a car is an exponential random variable with rate 1. If A.J.'s car and M.J.'s car are both brought in at time 0, with work starting on M.J.'s car only when A.J.'s car has been completely serviced, what is the probability that M.J.'s car is ready before time 2? The a...

0
Q: Investment in a treasury bill

ratrt13You invested $968 710 in a treasury bill with the face value of $1 000 000 with 91 days left till maturity. After 60 days you have the option to sell it for $989 250. Which option is more profitable? My solution: r1=(1000000-968710)/((91/360)*968710)=12,78% r2=(989250-968710)/((60/360)*968710...

0
Q: how to take the dual problem of a problem that has a sum in it?

ThijserI'm learning about dual problems and was trying to get to an understanding of how to take the dual of a problem that has a sum in it. For example if we try to optimize the sum of all values while keeping them under a given total max sum i=1...n(sixi) where sum i=1...n(pixi)<=p x>0 how wou...

0
Q: direct sum/products of groups

Nex_FriedrichWrite the following subgroups as a direct sum or direct product of $p$-groups: $$\text{a)}\ Z^{*}_{15},\ \ \text{b)}\ Z_{12} \times Z_{12} \times Z_{60}$$ where $Z^*{n}$ is a standard additive group modulo $n$ and $Z_{n}$ is multiplicative group modulo $n$.

0
Q: Game theory exam review question

Steven LiuThe following is a question from a past exam that I am studying: Two firms A and B make color and black and white television sets. Firm A can make either 200 color sets a week or 200 black and white sets. Firm B can make either 400 color or 200 black and white and 200 color, or 400 black and wh...

Welcome to Math.SE, Steven Liu. Words such as question are uninformative in titles. Please edit the title so that it better describes the specifics of your question. Do not hesitate to make it longer or include a formula if needed. More tips here. (from a bot)Normal Human 21 secs ago
0
Q: tricky statement about limits, TRUE or FALSE?

karhasIn a homework sheet with true or false questions I have found the folowing statement $\displaystyle \lim_{h \to 0}\left[ f(x+h)-f(x) \right]=0$ True OR False At the first shight of it True seems the right answer but then this came to my mind If $f$ is not continues at $x$ then $$\lim_{x ...

0
Q: non principal open affin

s1rr1I'm trying to find an open affine susbset whiche is not principal. the idea is to consider the ring $\displaystyle\frac{l[u,x,y,z]}{yz+uy^{2}+xz^{2}}$ (where $l$ a field) and show that $V:=D(y)\cup D(z)$ is non principal. Hartshorne has a criterion for affineness in the page 81 (ex 2.17). I hav...

0
Q: Difference between algebraic topology and geometric topology

user283871What are the main differences between these two areas? Does geometric topology in general, use more analytic techniques? Which one would most consider harder?

0
Q: converting toLogic Gate to NAND gates only

unreal1357what would the answer of this this be in the form of NAND only https://gyazo.com/3b3c87dbfe86203d032f8cf37fe0c44b

0
Q: ODE Modelling in Matlab

tom982I need to model human hairs in Matlab by solving ODEs, but I'm struggling to understand what to do. Here is all of the provided information: I've been asked to write a function returning the $(x,y,z)$ coordinates of the hairs given $L$, $R$, $f_x$ and a list of values for $\theta(L)$ and $\phi...

Short title. Tagged matlab. ODE Modelling in Matlab
0
Q: Limit of two-variable function

paduraI have big problem with the the limit of the following function: \lim_{(x,y)\to (0,0)} \left(\frac{x^2 y}{x^4 + y^2}\right) I have tried to estimate lower and upper bound for Three-Series Theorem and to to convert it to polar coordinates and it didn't work well :)

This site uses MathJax formatting of formulas. More tips here. (from a bot)Normal Human 21 secs ago
0
Q: Optimization problem, finding the dimensions of the container of least cost.

JackA closed rectangular container with a square base is to have a volume of 2000 cubic centimeters. It costs twice as much per square centimeter for the top and bottom as it does for the sides. Find the dimensions of the container of least cost. So the formula for the volume is: 20...

0
Q: Extension of $Z_2$ by $SO(n)$

Nex_FriedrichHow to show that the extension of group $Z_{2}$ by $SO(n)$: $$Id \rightarrow SO(n) \rightarrow O(n) \xrightarrow{det} Z_2 \rightarrow 1$$ is a direct product for odd $n$ and indirect product for even $n$.

0
Q: soving Newton's Law of Cooling with chi square distribution and excel solver

DaffyI find this tutorial for chi square distribution and excel solver and i dont now if i can use it for solving Newton's law of cooling ?

0
Q: I'd be grateful if somebody could help me with this probability problem.

Joseph RishikThe gross weekly sales at a certain restaurant are a normal random variable with mean $2200 and standard deviation $230. What is the probability that a. the total gross sales over the next 2 weeks exceeds $5000 b. weekly sales exceed $2200 in at least 2 of the next 3 weeks? Don't just give me ...

0
Q: Making equations dimentionless

darrenI have a equation of motion for a forced pendulum show below ((d^2*theta)/(dt^2)) = -(g/L)sin(theta) + C*cos(theta)sin(Dt) L=10cm, C=2(s^-2) and D=5(s^-1) I am trying to make this equation dimensionless by setting the follow equations (omega)^2 = g/L , beta=D/omega , gamma=c/(omega)^2...

This site uses MathJax formatting of formulas. More tips here. (autocomment)Normal Human 21 secs ago
0
Q: Compute an Integral1

Omar Oomaris there any one can helps me solve this problem of integral $$ \int \frac {x^2-y^2}{(x^2+y^2)^2} dy $$ ................................................................................................. ............................................................................................

Short title. Title ends with a digit. Compute an Integral1
0
Q: Oscillation of a function and continuity

luka5z I would be very grateful if somebody could verify my proof. Assume that $f$ is continuous at $p$. We can always choose $\delta_1>0, \delta_2>0$ such that $d'(f(x),f(p))\le\frac{1}{2n}$ whenever $d(x,p)\le\delta_1$ and $d'(f(y),f(p))\le\frac{1}{2n}$ whenever $d(y,p)\le\delta_2$. Set $\delta=\...

Tagged proof-verification. Oscillation of a function and continuity
0
Q: if $w$ is a 2-form, and $dw = 0$ what can we conclude?

FACEIT I have calculted $dw = D_1f_1 + D_2f_2 + D_3f_3 $, but I am not sure what I can conclude if the sum is equal to $0$. Anyhelp please.

Short question. Question contains please. if $w$ is a 2-form, and $dw = 0$ what can we conclude?
0
Q: Prove groups are isomorphic

Goose719Let p be a prime where $p$ $\neq$ 2. Prove $U(p) = U(2p)$ and $U(p^2)=\mathbb{Z}_p $ X $\mathbb{Z}_{p-1}$ I know $U(n)$ is the set of all numbers relatively prime to n. I know this statement is true, it is quite obvious, but I am having trouble proving. Can you help?

0
Q: Norm of $T^n$, where $Tf(x,y) = \begin{cases}f(x+y/b,y), &0<x<1-y/b,\\1/2f(x+y/b-1,y),& 1-y/b<x<1.\end{cases}$

xenLet $0 < a < b$ and $T\colon L^\infty((0,1)\times (a,b)) \to L^\infty((0,1)\times (a,b))$ be the operator defined by $$Tf(x,y) = \begin{cases}f(x+\frac yb,y), &0<x<1-\frac yb,\\\frac 12f(x+\frac yb-1,y),& 1-\frac yb<x<1,\end{cases}$$ where $x \in (0,1)$ and $y \in (a,b)$. Is it true that $$...

0
Q: Adding Bernstein sets

user114634Show by transfinite induction that there exists a Bernstein set $B$ such that $B + B$ = R, where $B + B$ = {$b_0 + b_1$ | $b_0$, $b_1 \in B$}. I believe I need to enumerate the family of Perfect sets, P such that ($B + B$) $\bigcap$ P $\ne$ $\emptyset$ and P $\not\subset$ $B + B$, as well as $B ...

0
Q: prove any sufficiently large family of k-sets has an L-intersecting subfamily of size n for some integer L.

user3316874I'n not really sure how I'm supposed to prove this. All I know is that I need to use Ramsey's Theorem.

0
Q: The Jones polynomial of the connected sum of two links.

Mohammed SabakI've been working on some knot invaririants and specialy the Jones Polynomial. I was able to prove that $ V_{K_1 \# K_2} = V_{K_1} V_{K_2} $ for two knots $ K_1 $ and $ K_2 $ . So I found my self asking if the same raltion holds in the case of links. All books states that indeed $ V_{L_1 \# L_2} ...

Questions tend to get more attention when they have a tag for a broad area of mathematics relevant to the question. Some of these tags might fit. (from a bot)Normal Human 20 secs ago
 
9:39 PM
0
Q: Probability with Combinations and Dice

IzzyA die is rolled five times. What is the probability of obtaining at most three 4’s?

0
Q: problem from Stoll's introduction to real analysis

1233211Suppose f: [a,b]$\rightarrow \Re $ is continuous. Let M=max{|f(x) : x$\in$[a,b]} Show that $$\lim_{n\rightarrow \infty}(\int_a^b |f(x)|^n)^{\frac{1}{n}}=M$$ My attempt: Suppose for contradiction that $\exists$ c$\in$[a,b] such that $$f(c)\gt \lim_{n\rightarrow \infty}(\int_a^b |f(x)|^n)^{\frac{...

Title contains problem. Tagged proof-writing. problem from Stoll's introduction to real analysis
0
Q: For which matrices $A \in \mathscr{M}_n(\mathbb C)$ is the similarity class of $A$ closed?

mathcounterexamples.netWhat are the matrices $A \in \mathscr{M}_n(\mathbb C)$ for which the similarity class is closed? What about the same question if we replace $\mathbb C$ by $\mathbb R$?

0
Q: Show that $L^p([a,b])\subset L^r([a,b])$ for any $1\leq r<p\leq\infty$.

Desperate FluffyShow that $L^p([a,b])\subset L^r([a,b])$ for any $1\leq r<p\leq\infty$. I thought this, or something similar, was a theorem somewhere, but now I can't find anything on it. If anyone knows of a link or something, please put it in a comment below. Now, a real question: how do I prove this? It s...

Question contains please. [Show that $L^p([a,b])\subset L^r([a,b])$ for any $1\leq r<p\leq\infty$.](math.stackexchange.com/q/1563175)
0
Q: seperating equations into ODEs

darrenI have a complex equation and need some help separating it into two first order differential equations. My equation is below (((d^2)*theta)/dt^2) = -sin(theta) + (gamma)*cos(theta)sin(betax) How do I separate this into two first order differential equations? I have no idea on the theory of t...

This site uses MathJax formatting of formulas. More tips here. (from a bot)Normal Human 20 secs ago
0
Q: $T(W_j) \subset W_j$

turtleWe have a linear operator $T: V \to V$. Suppose $W_j$ is the nullspace of $(T - \lambda_j I)^{r_j}$ for some $0 < r_j$, where $\lambda_j$ is a root of the minimal polynomial of $T$ of multiplicity $r_j$. I would like to show that $T(W_j) \subset W_j$. Here is my attempt: We want to show that $\...

A title should not be all-MathJax; having some plain text helps with search and navigation. (autocomment)Normal Human 20 secs ago
0
Q: Conditional expectation.Tthe expected number of patients with advanced stage cancer, given at least one patient has early stage cancer

user59036In a large population of patients, 20% have early stage cancer, 10% have advanced stage cancer, and the other 70% do not have cancer. Six patients from this population are randomly selected. Calculate the expected number of selected patients with advanced stage cancer, given that at least one of...

0
Q: Equivalence classes question

Bobo TD We define ∼: Z × Z as x ∼ y ⇔ |x − y| is divisible by 7. What is [1] ∼ and Z /∼ ? Is (13, 4) ∼ × ∼ (6, 9) where : X × X as x ∼ y ⇔ |x − y| is divisible by 5? Please explain.

Welcome to Math.SE, Bobo TD. Words such as question are uninformative in titles. Please edit the title so that it better describes the specifics of your question. Do not hesitate to make it longer or include a formula if needed. More tips here. (autocomment)Normal Human 20 secs ago
0
Q: For a finite cyclic group G with generator g and subgroup H where g^a is the smallest positive power of g in H, H contains all positive powers of g^a

user278055Am I right in thinking that since H is a cyclic subgroup, then h=g^a is a generator of H? Therefore it must contain all positive powers of h.

 
10:03 PM
Hello
f(x)=-10sin(2x)cos2x, where x is greater than or equal to 0 but smaller than or equal to pi.Write down an expression for f(x) in the form of k sin 4x
@NormalHuman how would you do that question?
 
0
Q: Damped motion direction

Dema UshchapovskyyI am getting very confused with signs of Damped Motion. The question goes like this: A light elastic string, of natural length $2a$ and modulus of elasticity $mg$, has a particle P of mass $m$ attached to its mod-point. One end of the string is attached to a fixed point A and the other end is a...

Questions tend to get more attention when they have a tag for a broad area of mathematics relevant to the question. Some of these tags might fit. (from a bot)Normal Human 20 secs ago
0
Q: derivative 0f a scalar field with respect to the modulus of the position vector

mhwk12I have the question: Given that f(r) is a twice differentiable scalar function of r = |r|, r = (x, y, z), and f′ = df/dr and f′′ = d2f/dr2 are its first and second derivatives, show that ∇ · ∇f(r) = f′′ + (2/r)*f′ I have been given the result that ∇f(r) =f′ * rhat (where rhat is the unit positio...

Welcome to Math.SE, mhwk12. This site uses MathJax formatting of formulas. More tips here. (autocomment)Normal Human 21 secs ago
0
Q: Is $2 = 123$ modulo $6$?

lucidgoldIs $2 = 123$ modulo $6$? How do I attack this question? Do I just answer: What is $123$ MOD $6$? The answer is 3. So: Is $2 = 123$ modulo $6$? Answer is NO!

Short title. Short question. Is $2 = 123$ modulo $6$?
 
10:23 PM
@NormalHuman hey uhhm are you there?
 
0
Q: Why is my $\epsilon - \delta$ proof incorrect?

ArthurProve that $f(x)= \sqrt{4+x^2}$ is continuous at $x_0$ using the $\epsilon -\delta$ definition of continuity. Textbook proof: $|\sqrt{4+x^2}-\sqrt{4+x^2}|=\frac{|4+x^2-(4+{x_0}^2)|}{\sqrt{4+x^2}+\sqrt{4+{x_0}^2}}=\frac{|x^2-{x_0}^2|}{\sqrt{4+x^2}+\sqrt{4+{x_0}^2}}=\underbrace{\frac{|x+x_0|}{\sq...

0
Q: Impossible Integration Problem

TomBeatrousWhat is the anti-derivative of x^(x-1)?

Short title. Title contains problem. Short question. Impossible Integration Problem
0
Q: Localization at a minimal prime in a Noetherian ring

MathmankIf I localize at a minimal prime ideal $p$ in a noetherian ring $A$, is $A_p$ finitely-generated as an $A$-algebra?

0
Q: Laplace equation in wave

user119144I was reading a paper having the following wave equation $ u_{tt} - \Delta u = 0 $ then multiplied by $ u_ {t} $ and is $\frac{d}{dt} \frac{1}{2} ||u_ {t} ||^2+||\Delta u ||^ 2 = 0 $ My problem is with the term or $ ||\Delta u ||^ 2$, I can not reach him, I tried using the definition of Lapl...

0
Q: Defined what is meant by a orthogonal projection from an inner product V to a subspace W of V

Aether WyjI cant find the answer from the textbook, can anyone help me with this... I don't know if this could be the answer: $$p= \frac {a•b}{a•a} a$$

0
Q: Find the upper and lower sums for $f\left(x\right)=\begin{cases} 1 & 1\leq x<3\\ 5 & x=3\\ 3 & 3<x\leq4 \end{cases} $

amandaFind the upper and lower sums for $f\left(x\right)=\begin{cases} 1 & 1\leq x<3\\ 5 & x=3\\ 3 & 3<x\leq4 \end{cases} $ Attempt $I_{1} = [1,3-a] \ with \ M_{1} = \sup_{I_{1}}f = 1 \ and \ m_{1} = \sup_{I_{1}}f = 1$ $I_{2} = [3-a,3+a] \ with\ M_{2} = \sup_{I_{2}}f = 5 \ and \ m_{2} = \sup_{I_{2}}f...

0
Q: Game theory exam review question

Steven LiuThe following is a question from a past exam that I am studying: For a 3-person game of perfect information. Let S denote the set {1,2,3}. First player A chooses i ϵ S. Then player B, knowing i chooses j ϵ S, j != i. Finally player C, knowing i and j, chooses k E S, k != j or i. The payoff gi...

Words such as question do not add information to titles. Please edit the title so that it better describes the specifics of your question. Do not hesitate to make it longer or include a formula if needed. More tips here. (autocomment)Normal Human 21 secs ago
0
Q: Representation of cosx as Taylor series

Dmitry KharatyanWhy the last term of cosx Tailor series is the o(x^2n+1) but not o(x^2n) ?

Consider replacing (analysis) with a more specific tag for the relevant branch of analysis. (autocomment)Normal Human 21 secs ago
 
10:55 PM
0
Q: Don't understand this proof of $(A \triangle B)\cup C = (A\cup C)\triangle (B\setminus C)$

user242007In this question, the most upvoted explanation of the identity in my title is this reply. I don't have the reputation to comment on the existing thread, so I'm asking here, because I am having a hard time following the explanation. I can prove this identity via Venn decomposition as in the checke...

0
Q: Show Galois extension with $[K : F] = n$ has an intermediate filed with $L$ with $[K : L] = p$, where $p$ is a prime divisor of $n$

MichaelIt is a two part question. Let $K/F$ be a Galois extension with $[K : F] = n$. If $p$ is a prime divisor of $n$, prove there is an intermediate field $L$ with $[K : L] = p$. Prove or disprove that there is an intermediate field $M$ with $[M : F] = p$. Could anyone help me for some hints please...

Question contains please. [Show Galois extension with $[K : F] = n$ has an intermediate filed with $L$ with $[K : L] = p$, where $p$ is a prime divisor of $n$](math.stackexchange.com/q/1563291)
0
Q: HOMEWORK - Proving that two linear maps are formally dual

mathcompI have this question in a homework sheet: So far, I have done parts $i), ii) $ and $iii)$ but I am really struggling with part $iv)$. Any help would be much appreiated as I have made no solid progress as of yet. Thank you

Title contains homework. Tagged proof-writing. HOMEWORK - Proving that two linear maps are formally dual
0
Q: Set of finite outer measure is contained in an open set of finite measure?

AndrewIn a proof I'm reading, the author remarks in the first line of the proof that a set of finite outer measure is contained in an open set of finite measure. I have spent some time thinking about this and it is not obvious to me that this is the case. Can someone please explain why this should be o...

0
Q: How many terms in the series arctan(x) would be needed to get pi to the 10th decimal place?

NaokiI got π=-4/3+4/5-4/7+4/9... but i can see that using this it will take me a very long time to reach the decimal expansion im looking for. i thought about setting (-1)^n (1/2n-1) less than .0000000001 but im not sure if that is a proper method.

This site uses MathJax formatting of formulas. More tips here. (autocomment)Normal Human 21 secs ago
 
11:10 PM
0
Q: How can I get two different questions merged?

Ashhar HasanThere are two questions. Both have the same final problem. How to get them merged? Also both the questions have a lot of too localized information that will need to be removed after the merge. What is the policy on that? Do we edit the resulting merge question and remove unnecessary information...

 
0
Q: Does Bayes theorem have two formulas?

Allie$$P(AlB)=P(A)P(B\mid A)/[P(A)P(B \mid A)+P(A^c)P(B \mid A^c)] $$ $$P(A \mid B) = [P(B \mid A)P(A)]/P(B)$$ 1) Are these two equations so called bayes formula?? 2) how do you distinguish whether to use the first one or the second one? 3) If i assume A and B are independent, and let P(A)=0.2, P...

Questions tend to get more attention when they have a tag for a broad area of mathematics relevant to the question. Some of these tags might fit. (autocomment)Normal Human 21 secs ago
 
0
Q: Is there a limit on how much rep I can give away?

FabbyNow that I've reached 25K, I'm not really interested in the next level: 100K is too far off and too much trouble for a T-shirt and a mug... So I've been giving away all my rep above 25K (sort of) to: Original answers Interesting answers Answers that helped me in the past Answers that took a lo...

 
11:29 PM
0
Q: Cohomologies of anticanonical sheaf of blow-up of $\mathbb{P}^2$ in 9 points

ARMLet $\pi:X\to\mathbb{P}^2$ be a blow-up of $\mathbb{P}^2$ in 9 points (in general position for example, but it doesn't matter). Then the canonical divisor of $X$ is equal $K_X=-3\pi^*H+\sum\limits_{k=1}^9E_k$, where $E_k$, $k=1,...,9$ are exceptional divisors. It is known and not hard to compute ...

0
Q: Elementary Combinatorics: A shop sells 5 kinds of cookies. How many ways to choose 16 if...

Hani Al-shafeiA cookie shop sells 5 different kinds of cookies. How many different ways are there to choose 16 cookies if... (a) you have no restrictions? (b) you pick at least two of each? (c) you pick at least 4 oatmeal cookies, at least 3 sugar cookies and at most 5 chocolate chip cookies? This is what...

0
Q: Choosing basis functions for least squares - f(x) = c0 + c1 sin x + c2 cos x

lehenI am kind of new in Math and I need to choose three basis functions g0, g1 and g2 for the least squares method (normal equations). Can anyone please first tell me what type of this equation is - f(x) = c0 + c1 sin x + c2 cos x (Polynomial/linear...I know neither is correct)(in which c0, c1 and c2...

Consider adding a tag for a broader subject area to which the question belongs. Some of these tags might fit. (from a bot)Normal Human 21 secs ago
0
Q: Radical of an ideal in K[X,Y,Z]

Diego HavezWhat is the radical of this ideal, $I = (X+Z, (Y-1)^{2}, Z^{2}-1+Y)$? I found that his radical is $(X,Y-1,Z)$, correct? Thank you!

Short question. [Radical of an ideal in K[X,Y,Z]](math.stackexchange.com/q/1563345)
0
Q: distance from a point to a subspace spanned by a set of vectors

MaryCan someone please help me with the following textbook problem? Find the closest point and the distance from b=(1, 1, 2, -2)T to the subspace spanned by (1, 2, -1, 0)T, (0, 1, -2, -1)T and (1, 0, 3, 2)T. I believe that I am supposed to use Gram matrix, but any help will be great. Thanks.

0
Q: What is the integral of this? BC 1999 problem 6.

HiDannyintegral of e^x/x^2 I can't for the life of me figure it out. Can anyone help?. Here's a link if you need to see it :P. http://apcentral.collegeboard.com/apc/members/repository/sg_calculus_bc_99.pdf

Title contains problem. Short question. What is the integral of this? BC 1999 problem 6.
 
11:58 PM
0
Q: phi divisors help

tvasileCompute ∅ (40), 𝜎(124), 𝑑(124) and check the equality in Σ∅(𝑑) = 40. Here's what I've done so far: Not really sure about the summation equality. ∅ (40) = ∅ (5) x ∅ (8) => 4x4 = 16 𝜎(124) = 1+2+4+1+62+124 = 224 𝑑(124) = 1,2,4,31,62,124 = 6 Σ∅(𝑑) = 40 => ??????????

Welcome to Math.SE, tvasile. Words such as help are uninformative in titles. Please edit the title so that it better describes the specifics of your question. Do not hesitate to make it longer or include a formula if needed. This site uses MathJax formatting of formulas. Questions tend to get more attention when they have a tag for a broad area of mathematics relevant to the question. Some of these tags might fit. More tips here. (from a bot)Normal Human 21 secs ago
 
00:00 - 16:0016:00 - 00:00

« first day (34 days earlier)      last day (540 days later) »